Difference between revisions of "2018 USAJMO Problems/Problem 2"
m (→Solution 1: minor clarification) |
Megahertz13 (talk | contribs) (→Problem) |
||
(One intermediate revision by one other user not shown) | |||
Line 1: | Line 1: | ||
== Problem == | == Problem == | ||
Let <math>a,b,c</math> be positive real numbers such that <math>a+b+c=4\sqrt[3]{abc}</math>. Prove that <cmath>2(ab+bc+ca)+4\min(a^2,b^2,c^2)\ge a^2+b^2+c^2.</cmath> | Let <math>a,b,c</math> be positive real numbers such that <math>a+b+c=4\sqrt[3]{abc}</math>. Prove that <cmath>2(ab+bc+ca)+4\min(a^2,b^2,c^2)\ge a^2+b^2+c^2.</cmath> | ||
+ | |||
+ | == Video Solution == | ||
+ | https://www.youtube.com/watch?v=eKIiaYNWUzM&t=5s | ||
==Solution 1== | ==Solution 1== | ||
WLOG let <math>a \leq b \leq c</math>. Add <math>2(ab+bc+ca)</math> to both sides of the inequality and factor to get: <cmath>4(a(a+b+c)+bc) \geq (a+b+c)^2</cmath> | WLOG let <math>a \leq b \leq c</math>. Add <math>2(ab+bc+ca)</math> to both sides of the inequality and factor to get: <cmath>4(a(a+b+c)+bc) \geq (a+b+c)^2</cmath> | ||
− | By substituting <math>a+b+c=\sqrt[3]{abc}</math>, we get: | + | By substituting <math>a+b+c=4\sqrt[3]{abc}</math>, we get: |
<cmath>\frac{4a\sqrt[3]{abc}+bc}{2} \geq 2\sqrt[3]{a^2b^2c^2}</cmath> | <cmath>\frac{4a\sqrt[3]{abc}+bc}{2} \geq 2\sqrt[3]{a^2b^2c^2}</cmath> | ||
The last inequality is true by AM-GM. Since all these steps are reversible, the proof is complete. | The last inequality is true by AM-GM. Since all these steps are reversible, the proof is complete. |
Latest revision as of 16:38, 5 August 2024
Problem
Let be positive real numbers such that . Prove that
Video Solution
https://www.youtube.com/watch?v=eKIiaYNWUzM&t=5s
Solution 1
WLOG let . Add to both sides of the inequality and factor to get: By substituting , we get: The last inequality is true by AM-GM. Since all these steps are reversible, the proof is complete.
Solution 2
WLOG let . Note that the equations are homogeneous, so WLOG let . Thus, the inequality now becomes , which simplifies to .
Now we will use the condition. Letting and , we have .
Plugging this into the inequality, we have , which is true since .
The problems on this page are copyrighted by the Mathematical Association of America's American Mathematics Competitions.
Solution 3
https://wiki-images.artofproblemsolving.com//6/69/IMG_8946.jpg
-srisainandan6
See also
2018 USAJMO (Problems • Resources) | ||
Preceded by Problem 1 |
Followed by Problem 3 | |
1 • 2 • 3 • 4 • 5 • 6 | ||
All USAJMO Problems and Solutions |